Question 1209773: If a_1, a_2, ..., a_19 satisfy
a_1 + a_2 + a_3 + a_4 = 1,
a_2 + a_3 + a_4 + a_5 = 2,
a_3 + a_4 + a_5 + a_6 = 3,
...
a_{16} + a_{17} + a_{18} + a_{19} = 16,
a_{17} + a_{18} + a_{19} + a_1 = 17,
a_{18} + a_{19} + a_1 + a_2 = 18,
a_{19} + a_1 + a_2 + a_3 = 19,
what is the value of a_19?
Found 2 solutions by CPhill, ikleyn: Answer by CPhill(1959) (Show Source):
You can put this solution on YOUR website! Let's analyze the given system of equations.
We have:
1. a_1 + a_2 + a_3 + a_4 = 1
2. a_2 + a_3 + a_4 + a_5 = 2
3. a_3 + a_4 + a_5 + a_6 = 3
...
16. a_16 + a_17 + a_18 + a_19 = 16
17. a_17 + a_18 + a_19 + a_1 = 17
18. a_18 + a_19 + a_1 + a_2 = 18
19. a_19 + a_1 + a_2 + a_3 = 19
Subtract equation (1) from equation (2):
(a_2 + a_3 + a_4 + a_5) - (a_1 + a_2 + a_3 + a_4) = 2 - 1
a_5 - a_1 = 1
a_5 = a_1 + 1
Subtract equation (2) from equation (3):
(a_3 + a_4 + a_5 + a_6) - (a_2 + a_3 + a_4 + a_5) = 3 - 2
a_6 - a_2 = 1
a_6 = a_2 + 1
In general, we have a_{n+4} = a_n + 1.
This means:
a_5 = a_1 + 1
a_9 = a_5 + 1 = a_1 + 2
a_13 = a_9 + 1 = a_1 + 3
a_17 = a_13 + 1 = a_1 + 4
Similarly,
a_6 = a_2 + 1
a_10 = a_6 + 1 = a_2 + 2
a_14 = a_10 + 1 = a_2 + 3
a_18 = a_14 + 1 = a_2 + 4
Also,
a_7 = a_3 + 1
a_11 = a_7 + 1 = a_3 + 2
a_15 = a_11 + 1 = a_3 + 3
a_19 = a_15 + 1 = a_3 + 4
And,
a_8 = a_4 + 1
a_12 = a_8 + 1 = a_4 + 2
a_16 = a_12 + 1 = a_4 + 3
Now let's use equations 1, 17, 18, and 19.
1. a_1 + a_2 + a_3 + a_4 = 1
17. a_17 + a_18 + a_19 + a_1 = 17
18. a_18 + a_19 + a_1 + a_2 = 18
19. a_19 + a_1 + a_2 + a_3 = 19
Substituting the expressions we found:
17. (a_1 + 4) + (a_2 + 4) + (a_3 + 4) + a_1 = 17
2a_1 + a_2 + a_3 + 12 = 17
2a_1 + a_2 + a_3 = 5
18. (a_2 + 4) + (a_3 + 4) + a_1 + a_2 = 18
a_1 + 2a_2 + a_3 + 8 = 18
a_1 + 2a_2 + a_3 = 10
19. (a_3 + 4) + a_1 + a_2 + a_3 = 19
a_1 + a_2 + 2a_3 + 4 = 19
a_1 + a_2 + 2a_3 = 15
From 1, a_4 = 1 - (a_1 + a_2 + a_3).
From 19, a_1 + a_2 + a_3 = 19 - a_19.
Substitute into 1:
19 - a_19 + a_4 = 1, so a_4 = a_19 - 18.
Since a_19 = a_3 + 4, a_3 = a_19 - 4.
From 1, a_1 + a_2 + a_3 = 1 - a_4 = 1 - (a_19 - 18) = 19 - a_19.
Now substitute a_3 = a_19 - 4 into a_1 + a_2 + 2a_3 = 15.
a_1 + a_2 + 2(a_19 - 4) = 15
a_1 + a_2 + 2a_19 - 8 = 15
a_1 + a_2 = 23 - 2a_19.
Substitute into a_1 + a_2 + a_3 = 19 - a_19.
23 - 2a_19 + a_19 - 4 = 19
19 - a_19 = 19
a_19 = 0.
Final Answer: The final answer is $\boxed{0}$
Answer by ikleyn(52786) (Show Source):
You can put this solution on YOUR website! .
If a_1, a_2, ..., a_19 satisfy
a_1 + a_2 + a_3 + a_4 = 1,
a_2 + a_3 + a_4 + a_5 = 2,
a_3 + a_4 + a_5 + a_6 = 3,
...
a_{16} + a_{17} + a_{18} + a_{19} = 16,
a_{17} + a_{18} + a_{19} + a_1 = 17,
a_{18} + a_{19} + a_1 + a_2 = 18,
a_{19} + a_1 + a_2 + a_3 = 19,
what is the value of a_19?
~~~~~~~~~~~~~~~~~~~~~~~~~~~~~~~~~~~~~~~~~
The solution and the final answer in the post by @CPhill are INCORRECT.
The idea is good and the implementation is good until some point.
After this point, implementation is wrong.
So, I will copy-paste the solution by @CPhill below in my post.
Then I will mark there the point till which the solution is correct.
After this point, I will place my calculations, and will complete the solution to the end.
Let's analyze the given system of equations.
We have:
1. a_1 + a_2 + a_3 + a_4 = 1
2. a_2 + a_3 + a_4 + a_5 = 2
3. a_3 + a_4 + a_5 + a_6 = 3
...
16. a_16 + a_17 + a_18 + a_19 = 16
17. a_17 + a_18 + a_19 + a_1 = 17
18. a_18 + a_19 + a_1 + a_2 = 18
19. a_19 + a_1 + a_2 + a_3 = 19
Subtract equation (1) from equation (2):
(a_2 + a_3 + a_4 + a_5) - (a_1 + a_2 + a_3 + a_4) = 2 - 1
a_5 - a_1 = 1
a_5 = a_1 + 1
Subtract equation (2) from equation (3):
(a_3 + a_4 + a_5 + a_6) - (a_2 + a_3 + a_4 + a_5) = 3 - 2
a_6 - a_2 = 1
a_6 = a_2 + 1
In general, we have a_{n+4} = a_n + 1.
This means:
a_5 = a_1 + 1
a_9 = a_5 + 1 = a_1 + 2
a_13 = a_9 + 1 = a_1 + 3
a_17 = a_13 + 1 = a_1 + 4
Similarly,
a_6 = a_2 + 1
a_10 = a_6 + 1 = a_2 + 2
a_14 = a_10 + 1 = a_2 + 3
a_18 = a_14 + 1 = a_2 + 4
Also,
a_7 = a_3 + 1
a_11 = a_7 + 1 = a_3 + 2
a_15 = a_11 + 1 = a_3 + 3
a_19 = a_15 + 1 = a_3 + 4
And,
a_8 = a_4 + 1
a_12 = a_8 + 1 = a_4 + 2
a_16 = a_12 + 1 = a_4 + 3
Now let's use equations 1, 17, 18, and 19.
From eq. 1 --> a_1 + a_2 + a_3 + a_4 = 1
From eq.17 --> a_17 + a_18 + a_19 + a_1 = 17
From eq.18 --> a_18 + a_19 + a_1 + a_2 = 18
From eq.19 --> a_19 + a_1 + a_2 + a_3 = 19 (*)
Substituting the expressions we found:
17. (a_1 + 4) + (a_2 + 4) + (a_3 + 4) + a_1 = 17
2a_1 + a_2 + a_3 + 12 = 17
2a_1 + a_2 + a_3 = 5 (17)
18. (a_2 + 4) + (a_3 + 4) + a_1 + a_2 = 18
a_1 + 2a_2 + a_3 + 8 = 18
a_1 + 2a_2 + a_3 = 10 (18)
19. (a_3 + 4) + a_1 + a_2 + a_3 = 19
a_1 + a_2 + 2a_3 + 4 = 19
a_1 + a_2 + 2a_3 = 15 (19)
<<<---=== till this point, the solution by @CPhill is correct.
After this point, it is WRONG.
<<<---=== So, starting from this point, my solution is going.
20. Add equations (17), (18) and (19). You will get
4a_1 + 4a_2 + 4a_3 = 5 + 10 + 15
4a_1 + 4a_2 + 4a_3 = 30.
Divide both sides by 4
a_1 + a_2 + a_3 = 7.5. (20)
Now we are on the finish line.
From equation (17) subtract equation (20). You will get a_1 = 5 - 7.5 = -2.5.
From equation (18) subtract equation (20). You will get a_2 = 10 - 7.5 = 2.5.
From equation (19) subtract equation (20). You will get a_3 = 15 - 7.5 = 7.5.
From equation (*), a_19 = 19 - a_1 - a_2 - a_3 = 19 - 0 - (-2.5) - 2.5 - 7.5 = 11.5
For completeness, let's determine a_1 from the very first equation in this post
a_1 = 1 - (a_2 + a_3 + a_4) = 1 - ((-2.5) + 2.5 + 7.5) = 1 - 7.5 = -6.5.
From this point, all 19 terms a_1, a_2, a_3, a_4, a_5, . . . , a_19 can be determined.
See this table below
a_1 a_2 a_3 a_4 a_5 a_6 a_7 a_8 a_9 a_10 a_11 a_12 a_13 a_14 a_15 a_16 a_17 a_18 a_19
-2.5 2.5 7.5 -6.5 -1.5 3.5 8.5 -5.5 -0.5 4.5 9.5 -4.5 0.5 5.5 10.5 5.5 1.5 6.5 11.5
You may check that all given 19 original equations are satisfied.
Final Answer: The final answer is a_19 = 11.5.
Solved completely and correctly.
//////////////////////////////////
By the way, I looked at the solution by Google AI for this problem of today, 03/02/25. The link to it is
https://www.google.com/search?q=If+a_1%2C+a_2%2C+...%2C+a_19+satisfy+a_1+%2B+a_2+%2B+a_3+%2B+a_4+%3D+1%2C+a_2+%2B+a_3+%2B+a_4+%2B+a_5+%3D+2%2C+a_3+%2B+a_4+%2B+a_5+%2B+a_6+%3D+3%2C+...+a_%7B16%7D+%2B+a_%7B17%7D+%2B+a_%7B18%7D+%2B+a_%7B19%7D+%3D+16%2C+a_%7B17%7D+%2B+a_%7B18%7D+%2B+a_%7B19%7D+%2B+a_1+%3D+17%2C+a_%7B18%7D+%2B+a_%7B19%7D+%2B+a_1+%2B+a_2+%3D+18%2C+a_%7B19%7D+%2B+a_1+%2B+a_2+%2B+a_3+%3D+19%2C+what+is+the+value+of+a_19%3F&rlz=1C1CHBF_enUS1071US1071&oq=If+a_1%2C+a_2%2C+...%2C+a_19+satisfy+++a_1+%2B+a_2+%2B+a_3+%2B+a_4+%3D+1%2C++a_2+%2B+a_3+%2B+a_4+%2B+a_5+%3D+2%2C++a_3+%2B+a_4+%2B+a_5+%2B+a_6+%3D+3%2C+++...++a_%7B16%7D+%2B+a_%7B17%7D+%2B+a_%7B18%7D+%2B+a_%7B19%7D+%3D+16%2C+a_%7B17%7D+%2B+a_%7B18%7D+%2B+a_%7B19%7D+%2B+a_1+%3D+17%2C++a_%7B18%7D+%2B+a_%7B19%7D+%2B+a_1+%2B+a_2+%3D+18%2C++a_%7B19%7D+%2B+a_1+%2B+a_2+%2B+a_3+%3D+19%2C+++what+is+the+value+of+a_19%3F&gs_lcrp=EgZjaHJvbWUyBggAEEUYOdIBCTE5NjZqMGoxNagCCLACAfEFoMFf_ttl--Y&sourceid=chrome&ie=UTF-8
This solution was slightly different from that by @CPhill, but also was WRONG.
Naturally, I reported them about their wrong solution.
Hope they will fix their artificial mind.
|
|
|